Eigenvalues of sum of a matrix and its inverseEigenvalues of a matrix and its squarecomplex eigenvalues and invariant spacesProperties of a matrix that shares the set of real eigenvalues with its inverseEigenvalues of an upper Hessenberg matrixEigenvalues of complex special orthogonal matrix$A in M(n,mathbb R)$ has all its eigenvalues real , then is it true that all the eigenvalues of $A^2-A$ are also real?What are the eigenvalues of the following symmetric matrix?What is a necessary and sufficient condition for all the eigenvalues of a non-symmetric matrix to be positive?Is there any matrix with this propery?Eigenvalues of a square matrix

How can I remove material from this wood beam?

What is the logic behind taxing money for property?

Next date with distinct digits

How to safely destroy (a large quantity of) valid checks?

Why am I Seeing A Weird "Notch" on the Data Line For Some Logical 1s?

How to communicate to my GM that not being allowed to use stealth isn't fun for me?

Is it expected that a reader will skip parts of what you write?

Does Google Sheets allow moving a chart to its own sheet?

What would be the way to say "just saying" in German? (Not the literal translation)

Write a function that checks if a string starts with or contains something

Ability To Change Root User Password (Vulnerability?)

Can I utilise a baking stone to make crepes?

What standard algorithm can determine if exactly one of a container satisfies a predicate?

Is it possible for a vehicle to be manufactured without a catalytic converter?

Why Does Mama Coco Look Old After Going to the Other World?

Are polynomials with the same roots identical?

Who won a Game of Bar Dice?

Is there a set of positive integers of density 1 which contains no infinite arithmetic progression?

How to “listen” to existing circuit

What aircraft was used as Air Force One for the flight between Southampton and Shannon?

Will Roalesk, Apex Hybrid, trigger Sharktocrab twice?

New bike, tubeless tire will not inflate

Can the removal of a duty-free sales trolley result in a measurable reduction in emissions?

What are neighboring ports?



Eigenvalues of sum of a matrix and its inverse


Eigenvalues of a matrix and its squarecomplex eigenvalues and invariant spacesProperties of a matrix that shares the set of real eigenvalues with its inverseEigenvalues of an upper Hessenberg matrixEigenvalues of complex special orthogonal matrix$A in M(n,mathbb R)$ has all its eigenvalues real , then is it true that all the eigenvalues of $A^2-A$ are also real?What are the eigenvalues of the following symmetric matrix?What is a necessary and sufficient condition for all the eigenvalues of a non-symmetric matrix to be positive?Is there any matrix with this propery?Eigenvalues of a square matrix













2












$begingroup$


Let $M$ be a $ntimes n$ matrix such that $M^3=I$.Suppose that $Mv neq v$ for any non zero vector $v$.Then which of the following isare true?




1).$M+M^-1$ has real eigenvalues




The solution i tried is



Here it is given that $M^3=I$ so from here it is confirm that its minimal polynomial can be from $(x-1),(x^2+x+1) ;or; (x-1)(x^2+x+1)$, but according to given condition $(x-1); and ;(x-1);(x^2+x+1)$ can't be minimal polynomial.So the only possibility is $(x^2+x+1)$



From above it is confirmed that roots are complex roots ,but still i have no idea how to prove $M+M^-1$ has real eigenvalue



please help!



Thankyou










share|cite|improve this question









New contributor



Honey Kumar is a new contributor to this site. Take care in asking for clarification, commenting, and answering.
Check out our Code of Conduct.






$endgroup$
















    2












    $begingroup$


    Let $M$ be a $ntimes n$ matrix such that $M^3=I$.Suppose that $Mv neq v$ for any non zero vector $v$.Then which of the following isare true?




    1).$M+M^-1$ has real eigenvalues




    The solution i tried is



    Here it is given that $M^3=I$ so from here it is confirm that its minimal polynomial can be from $(x-1),(x^2+x+1) ;or; (x-1)(x^2+x+1)$, but according to given condition $(x-1); and ;(x-1);(x^2+x+1)$ can't be minimal polynomial.So the only possibility is $(x^2+x+1)$



    From above it is confirmed that roots are complex roots ,but still i have no idea how to prove $M+M^-1$ has real eigenvalue



    please help!



    Thankyou










    share|cite|improve this question









    New contributor



    Honey Kumar is a new contributor to this site. Take care in asking for clarification, commenting, and answering.
    Check out our Code of Conduct.






    $endgroup$














      2












      2








      2





      $begingroup$


      Let $M$ be a $ntimes n$ matrix such that $M^3=I$.Suppose that $Mv neq v$ for any non zero vector $v$.Then which of the following isare true?




      1).$M+M^-1$ has real eigenvalues




      The solution i tried is



      Here it is given that $M^3=I$ so from here it is confirm that its minimal polynomial can be from $(x-1),(x^2+x+1) ;or; (x-1)(x^2+x+1)$, but according to given condition $(x-1); and ;(x-1);(x^2+x+1)$ can't be minimal polynomial.So the only possibility is $(x^2+x+1)$



      From above it is confirmed that roots are complex roots ,but still i have no idea how to prove $M+M^-1$ has real eigenvalue



      please help!



      Thankyou










      share|cite|improve this question









      New contributor



      Honey Kumar is a new contributor to this site. Take care in asking for clarification, commenting, and answering.
      Check out our Code of Conduct.






      $endgroup$




      Let $M$ be a $ntimes n$ matrix such that $M^3=I$.Suppose that $Mv neq v$ for any non zero vector $v$.Then which of the following isare true?




      1).$M+M^-1$ has real eigenvalues




      The solution i tried is



      Here it is given that $M^3=I$ so from here it is confirm that its minimal polynomial can be from $(x-1),(x^2+x+1) ;or; (x-1)(x^2+x+1)$, but according to given condition $(x-1); and ;(x-1);(x^2+x+1)$ can't be minimal polynomial.So the only possibility is $(x^2+x+1)$



      From above it is confirmed that roots are complex roots ,but still i have no idea how to prove $M+M^-1$ has real eigenvalue



      please help!



      Thankyou







      linear-algebra eigenvalues-eigenvectors linear-transformations






      share|cite|improve this question









      New contributor



      Honey Kumar is a new contributor to this site. Take care in asking for clarification, commenting, and answering.
      Check out our Code of Conduct.










      share|cite|improve this question









      New contributor



      Honey Kumar is a new contributor to this site. Take care in asking for clarification, commenting, and answering.
      Check out our Code of Conduct.








      share|cite|improve this question




      share|cite|improve this question








      edited Jun 2 at 10:50









      José Carlos Santos

      190k24146264




      190k24146264






      New contributor



      Honey Kumar is a new contributor to this site. Take care in asking for clarification, commenting, and answering.
      Check out our Code of Conduct.








      asked Jun 2 at 10:33









      Honey KumarHoney Kumar

      274




      274




      New contributor



      Honey Kumar is a new contributor to this site. Take care in asking for clarification, commenting, and answering.
      Check out our Code of Conduct.




      New contributor




      Honey Kumar is a new contributor to this site. Take care in asking for clarification, commenting, and answering.
      Check out our Code of Conduct.






















          2 Answers
          2






          active

          oldest

          votes


















          3












          $begingroup$

          If $x^2+x+1$ is the minimal polynomial of $M$ then $M^2+M+I=0$. Also $M^3=I$ implies $M^-1=M^2$.
          Hence $$M^-1+M=M^2+M=-I$$.






          share|cite|improve this answer








          New contributor



          Naman is a new contributor to this site. Take care in asking for clarification, commenting, and answering.
          Check out our Code of Conduct.





          $endgroup$




















            2












            $begingroup$

            Let $lambda$ be an eigenvalue of $M$. Then, since $M^3=operatornameId$, $lambda^3=1$. Butbeginalignlambda^3=1&iff(lambda-1)(lambda^2+lambda+1)=0\&ifflambda^2+lambda+1=0,endalignsince $lambdaneq1$. There are only two numbers $lambda$ such that $lambda^2+lambda+1=0$: $-frac12pmfracsqrt32i$. In each case, $frac1lambda=overlinelambda$. If $v$ is an eigenvector corresponding to the eigenvalue $lambda$, then$$(M+M^-1).v=lambda v+frac1lambda v=left(lambda+overlinelambdaright)v=2operatornameRe(lambda)v=-v.$$






            share|cite|improve this answer











            $endgroup$













              Your Answer








              StackExchange.ready(function()
              var channelOptions =
              tags: "".split(" "),
              id: "69"
              ;
              initTagRenderer("".split(" "), "".split(" "), channelOptions);

              StackExchange.using("externalEditor", function()
              // Have to fire editor after snippets, if snippets enabled
              if (StackExchange.settings.snippets.snippetsEnabled)
              StackExchange.using("snippets", function()
              createEditor();
              );

              else
              createEditor();

              );

              function createEditor()
              StackExchange.prepareEditor(
              heartbeatType: 'answer',
              autoActivateHeartbeat: false,
              convertImagesToLinks: true,
              noModals: true,
              showLowRepImageUploadWarning: true,
              reputationToPostImages: 10,
              bindNavPrevention: true,
              postfix: "",
              imageUploader:
              brandingHtml: "Powered by u003ca class="icon-imgur-white" href="https://imgur.com/"u003eu003c/au003e",
              contentPolicyHtml: "User contributions licensed under u003ca href="https://creativecommons.org/licenses/by-sa/3.0/"u003ecc by-sa 3.0 with attribution requiredu003c/au003e u003ca href="https://stackoverflow.com/legal/content-policy"u003e(content policy)u003c/au003e",
              allowUrls: true
              ,
              noCode: true, onDemand: true,
              discardSelector: ".discard-answer"
              ,immediatelyShowMarkdownHelp:true
              );



              );






              Honey Kumar is a new contributor. Be nice, and check out our Code of Conduct.









              draft saved

              draft discarded


















              StackExchange.ready(
              function ()
              StackExchange.openid.initPostLogin('.new-post-login', 'https%3a%2f%2fmath.stackexchange.com%2fquestions%2f3248421%2feigenvalues-of-sum-of-a-matrix-and-its-inverse%23new-answer', 'question_page');

              );

              Post as a guest















              Required, but never shown

























              2 Answers
              2






              active

              oldest

              votes








              2 Answers
              2






              active

              oldest

              votes









              active

              oldest

              votes






              active

              oldest

              votes









              3












              $begingroup$

              If $x^2+x+1$ is the minimal polynomial of $M$ then $M^2+M+I=0$. Also $M^3=I$ implies $M^-1=M^2$.
              Hence $$M^-1+M=M^2+M=-I$$.






              share|cite|improve this answer








              New contributor



              Naman is a new contributor to this site. Take care in asking for clarification, commenting, and answering.
              Check out our Code of Conduct.





              $endgroup$

















                3












                $begingroup$

                If $x^2+x+1$ is the minimal polynomial of $M$ then $M^2+M+I=0$. Also $M^3=I$ implies $M^-1=M^2$.
                Hence $$M^-1+M=M^2+M=-I$$.






                share|cite|improve this answer








                New contributor



                Naman is a new contributor to this site. Take care in asking for clarification, commenting, and answering.
                Check out our Code of Conduct.





                $endgroup$















                  3












                  3








                  3





                  $begingroup$

                  If $x^2+x+1$ is the minimal polynomial of $M$ then $M^2+M+I=0$. Also $M^3=I$ implies $M^-1=M^2$.
                  Hence $$M^-1+M=M^2+M=-I$$.






                  share|cite|improve this answer








                  New contributor



                  Naman is a new contributor to this site. Take care in asking for clarification, commenting, and answering.
                  Check out our Code of Conduct.





                  $endgroup$



                  If $x^2+x+1$ is the minimal polynomial of $M$ then $M^2+M+I=0$. Also $M^3=I$ implies $M^-1=M^2$.
                  Hence $$M^-1+M=M^2+M=-I$$.







                  share|cite|improve this answer








                  New contributor



                  Naman is a new contributor to this site. Take care in asking for clarification, commenting, and answering.
                  Check out our Code of Conduct.








                  share|cite|improve this answer



                  share|cite|improve this answer






                  New contributor



                  Naman is a new contributor to this site. Take care in asking for clarification, commenting, and answering.
                  Check out our Code of Conduct.








                  answered Jun 2 at 10:43









                  NamanNaman

                  3207




                  3207




                  New contributor



                  Naman is a new contributor to this site. Take care in asking for clarification, commenting, and answering.
                  Check out our Code of Conduct.




                  New contributor




                  Naman is a new contributor to this site. Take care in asking for clarification, commenting, and answering.
                  Check out our Code of Conduct.























                      2












                      $begingroup$

                      Let $lambda$ be an eigenvalue of $M$. Then, since $M^3=operatornameId$, $lambda^3=1$. Butbeginalignlambda^3=1&iff(lambda-1)(lambda^2+lambda+1)=0\&ifflambda^2+lambda+1=0,endalignsince $lambdaneq1$. There are only two numbers $lambda$ such that $lambda^2+lambda+1=0$: $-frac12pmfracsqrt32i$. In each case, $frac1lambda=overlinelambda$. If $v$ is an eigenvector corresponding to the eigenvalue $lambda$, then$$(M+M^-1).v=lambda v+frac1lambda v=left(lambda+overlinelambdaright)v=2operatornameRe(lambda)v=-v.$$






                      share|cite|improve this answer











                      $endgroup$

















                        2












                        $begingroup$

                        Let $lambda$ be an eigenvalue of $M$. Then, since $M^3=operatornameId$, $lambda^3=1$. Butbeginalignlambda^3=1&iff(lambda-1)(lambda^2+lambda+1)=0\&ifflambda^2+lambda+1=0,endalignsince $lambdaneq1$. There are only two numbers $lambda$ such that $lambda^2+lambda+1=0$: $-frac12pmfracsqrt32i$. In each case, $frac1lambda=overlinelambda$. If $v$ is an eigenvector corresponding to the eigenvalue $lambda$, then$$(M+M^-1).v=lambda v+frac1lambda v=left(lambda+overlinelambdaright)v=2operatornameRe(lambda)v=-v.$$






                        share|cite|improve this answer











                        $endgroup$















                          2












                          2








                          2





                          $begingroup$

                          Let $lambda$ be an eigenvalue of $M$. Then, since $M^3=operatornameId$, $lambda^3=1$. Butbeginalignlambda^3=1&iff(lambda-1)(lambda^2+lambda+1)=0\&ifflambda^2+lambda+1=0,endalignsince $lambdaneq1$. There are only two numbers $lambda$ such that $lambda^2+lambda+1=0$: $-frac12pmfracsqrt32i$. In each case, $frac1lambda=overlinelambda$. If $v$ is an eigenvector corresponding to the eigenvalue $lambda$, then$$(M+M^-1).v=lambda v+frac1lambda v=left(lambda+overlinelambdaright)v=2operatornameRe(lambda)v=-v.$$






                          share|cite|improve this answer











                          $endgroup$



                          Let $lambda$ be an eigenvalue of $M$. Then, since $M^3=operatornameId$, $lambda^3=1$. Butbeginalignlambda^3=1&iff(lambda-1)(lambda^2+lambda+1)=0\&ifflambda^2+lambda+1=0,endalignsince $lambdaneq1$. There are only two numbers $lambda$ such that $lambda^2+lambda+1=0$: $-frac12pmfracsqrt32i$. In each case, $frac1lambda=overlinelambda$. If $v$ is an eigenvector corresponding to the eigenvalue $lambda$, then$$(M+M^-1).v=lambda v+frac1lambda v=left(lambda+overlinelambdaright)v=2operatornameRe(lambda)v=-v.$$







                          share|cite|improve this answer














                          share|cite|improve this answer



                          share|cite|improve this answer








                          edited Jun 2 at 10:54









                          P. Quinton

                          2,2301217




                          2,2301217










                          answered Jun 2 at 10:45









                          José Carlos SantosJosé Carlos Santos

                          190k24146264




                          190k24146264




















                              Honey Kumar is a new contributor. Be nice, and check out our Code of Conduct.









                              draft saved

                              draft discarded


















                              Honey Kumar is a new contributor. Be nice, and check out our Code of Conduct.












                              Honey Kumar is a new contributor. Be nice, and check out our Code of Conduct.











                              Honey Kumar is a new contributor. Be nice, and check out our Code of Conduct.














                              Thanks for contributing an answer to Mathematics Stack Exchange!


                              • Please be sure to answer the question. Provide details and share your research!

                              But avoid


                              • Asking for help, clarification, or responding to other answers.

                              • Making statements based on opinion; back them up with references or personal experience.

                              Use MathJax to format equations. MathJax reference.


                              To learn more, see our tips on writing great answers.




                              draft saved


                              draft discarded














                              StackExchange.ready(
                              function ()
                              StackExchange.openid.initPostLogin('.new-post-login', 'https%3a%2f%2fmath.stackexchange.com%2fquestions%2f3248421%2feigenvalues-of-sum-of-a-matrix-and-its-inverse%23new-answer', 'question_page');

                              );

                              Post as a guest















                              Required, but never shown





















































                              Required, but never shown














                              Required, but never shown












                              Required, but never shown







                              Required, but never shown

































                              Required, but never shown














                              Required, but never shown












                              Required, but never shown







                              Required, but never shown







                              Popular posts from this blog

                              Get product attribute by attribute group code in magento 2get product attribute by product attribute group in magento 2Magento 2 Log Bundle Product Data in List Page?How to get all product attribute of a attribute group of Default attribute set?Magento 2.1 Create a filter in the product grid by new attributeMagento 2 : Get Product Attribute values By GroupMagento 2 How to get all existing values for one attributeMagento 2 get custom attribute of a single product inside a pluginMagento 2.3 How to get all the Multi Source Inventory (MSI) locations collection in custom module?Magento2: how to develop rest API to get new productsGet product attribute by attribute group code ( [attribute_group_code] ) in magento 2

                              Category:9 (number) SubcategoriesMedia in category "9 (number)"Navigation menuUpload mediaGND ID: 4485639-8Library of Congress authority ID: sh85091979ReasonatorScholiaStatistics

                              Magento 2.3: How do i solve this, Not registered handle, on custom form?How can i rewrite TierPrice Block in Magento2magento 2 captcha not rendering if I override layout xmlmain.CRITICAL: Plugin class doesn't existMagento 2 : Problem while adding custom button order view page?Magento 2.2.5: Overriding Admin Controller sales/orderMagento 2.2.5: Add, Update and Delete existing products Custom OptionsMagento 2.3 : File Upload issue in UI Component FormMagento2 Not registered handleHow to configured Form Builder Js in my custom magento 2.3.0 module?Magento 2.3. How to create image upload field in an admin form